1962 AHSME Problems/Problem 14

Revision as of 22:45, 9 November 2013 by Fadebekun (talk | contribs) (Created page with "==Problem== Let <math>s</math> be the limiting sum of the geometric series <math>4- \frac83 + \frac{16}{9} - \dots</math>, as the number of terms increases without bound. Then <m...")
(diff) ← Older revision | Latest revision (diff) | Newer revision → (diff)

Problem

Let $s$ be the limiting sum of the geometric series $4- \frac83 + \frac{16}{9} - \dots$, as the number of terms increases without bound. Then $s$ equals:

$\textbf{(A)}\ \text{a number between 0 and 1}\qquad\textbf{(B)}\ 2.4\qquad\textbf{(C)}\ 2.5\qquad\textbf{(D)}\ 3.6\qquad\textbf{(E)}\ 12$

Solution

"Unsolved"